Laurentvidal.fr est la solution idéale pour ceux qui recherchent des réponses rapides et précises à leurs questions. Découvrez la facilité d'obtenir des réponses rapides et précises à vos questions grâce à l'aide de professionnels sur notre plateforme. Explorez des solutions complètes à vos questions grâce à une large gamme de professionnels sur notre plateforme conviviale.

Bonjour,pourriez vous m’aidez pour cet exercice s’il vous plaît, merci d’avance.

Bonjourpourriez Vous Maidez Pour Cet Exercice Sil Vous Plaît Merci Davance class=

Sagot :

Tenurf

Réponse :

bjr

Explications étape par étape

Tout d abord il est bon de se rappeler les formules de derivation

pour la fonction f telle que

f(x) = k (k constante reelle) alors f'(x)=0

f(x) = x alors f'(x)=1

f(x) = ax+b alors f'(x)=a

f(x) = ax^n alors f'(x)=anx^(n-1)

je note ^ pour la puissance

x^n veut dire x puissance n

et toutes les autres qui se trouvent dans ton cours

Du coup pour calculer les derivees demandees il faut appliquer les formules

De ce fait

f(x) = 2x-5 donne

f'(x) = 2

et f'(1)=f'(-1)=2

g(x) = -x^2+3x+1 donc

g'(x) = -2x+3

et  

g'(1) = - 2 + 3  = 1

g'(-1) = 2+3 = 5

h(x) = 5x^3-2x

h'(x) = 15x^2-2

donc  

h'(1) = 15 - 2 = 13

et

h'(-2) = 15 * 4 - 2 = 60 - 2 = 58

Réponse :

Il suffit p'appliquer la formule

f'(a)=lim qd h tend vers 0 de [f(a+h)-f(a)]/h

Explications étape par étape

je te fais la dernière essaie de faire les deux premières

il faut connaître les identités remarquables

(a+b)³=a³+3a²b+3ab²+b³ et (a-b)³=a³-3a²b+3ab²-b³

                                     **********************

k(x)=5x³-2x je la nomme k(x) pour ne pas confondre avec l'accroissement h

k'(1)=lim qd h tend vers 0 de [k(1+h)-k(1)]/h

=[5(1+h)³-2(1+h)-5(1)³+2(1)]/h

=[5(1³+3h+3h²+h³)-2-2h-5-2]/h=(5+15h+15h²+5h³-2h-5)/h

=h(5h²+15+15h-2)/h

Après simplification par h il reste

k'(1)=lim qd h tend vers 0 de 5h²+15h+15-2=13

donc k'(1)=13

                               *******************

k'(-2)=lim qd h tend vers 0 de [k(-2+h)-k(-2)]/h

=[5(-2+h)³-2(-2+h)-5(-2)³+2(-2)]/h

=[5(-8+12h-12h²+h³)+4-2h+40-4]/h=(60h-60h²+5h³-2h)/h

Après simplification par h il reste

k'(-2)=lim qd h tend vers 0 de 60+60h+5h²-2=58

donc k'(-2)=58

                                  *********************

Vérifications  k(x)=5x³-2x  sa dérivée est   k'(x)=15x²-2

k'(1)=15-2=13 et k'(-2)=60-2=58